Difference between revisions of "2021 JMPSC Accuracy Problems/Problem 7"

(Solution)
 
Line 3: Line 3:
  
 
==Solution==
 
==Solution==
Notice that <math>C</math> can only be <math>0</math> and <math>5</math>. However, <math>790</math> is not divisible by <math>3</math>, so <cmath>3 \times ABC = 795</cmath> <cmath>ABC = 265</cmath> Thus, <math>3A + 2B + C = \boxed{23}</math>
+
Notice that <math>C</math> can only be <math>0</math>, <math>1</math>, and <math>5</math>. However, <math>790</math> and <math>791</math> are not divisible by <math>3</math>, so <cmath>3 \times ABC = 795</cmath> <cmath>ABC = 265</cmath> Thus, <math>3A + 2B + C = \boxed{23}</math>
  
 
~Bradygho
 
~Bradygho

Latest revision as of 10:42, 18 July 2021

Problem

If $A$, $B$, and $C$ each represent a single digit and they satisfy the equation \[\begin{array}{cccc}& A & B & C \\ \times & &  &3 \\ \hline  & 7 & 9 & C\end{array},\] find $3A+2B+C$.

Solution

Notice that $C$ can only be $0$, $1$, and $5$. However, $790$ and $791$ are not divisible by $3$, so \[3 \times ABC = 795\] \[ABC = 265\] Thus, $3A + 2B + C = \boxed{23}$

~Bradygho

Solution 2

Clearly we see $C=1$ does not work, but $C=5$ works with simple guess-and-check. We have $AB5=\frac{795}{3}=265$, so $A=2$ and $B=6$. The answer is $3(2)+6(2)+1(5)=\boxed{23}$

~Geometry285

Solution 3

Easily, we can see that $A=2$. Therefore,\[\overline{BC} \cdot 3 = \overline{19C}.\]We can see that $C$ must be $1$ or $5$. If $C=1$, then\[\overline{B1} \cdot 3 = 191.\]This doesn't work because $191$ isn't divisible by $3$. If $C=5$, then\[\overline{B5} \cdot 3 = 195.\]Therefore, $B=6$. So, we have $3(2) + 2(6) + 5=6+12+5=18+5=\boxed{23}$.

- kante314 -

Solution 4

Notice that the only values of $C$ that have $3C = 10n+C$ for some $n$ are $0$ and $5$. If $C=0$, then we have $AB0 \cdot 3 = 790$, and so $AB \cdot 3 = 79$. Notice that $79$ is not divisible by $3$, so $C=0$ is not a valid solution. Next, when $C=5$, we have that $AB5 \cdot 3 = 795$. Solving for $A$ and $B$ tells us that $A=2$ and $B=6$, so the answer is $3 \cdot 2 + 2 \cdot 6 + 5 = 6 + 12 + 5 = \boxed{23}$.

~Mathdreams

See also

  1. Other 2021 JMPSC Accuracy Problems
  2. 2021 JMPSC Accuracy Answer Key
  3. All JMPSC Problems and Solutions

The problems on this page are copyrighted by the Junior Mathematicians' Problem Solving Competition. JMPSC.png